Ein Tensorprodukt zweier Spin-1-Teilchen

Ich bin ziemlich verwirrt und hatte gehofft, dass mir jemand helfen könnte, dieses (wahrscheinlich eher elementare) Problem zu lösen. Ich habe zwei Teilchen mit Spin 1, deren Zustand ich beschreibe M S Und M ICH bzw. Beide können die Werte -1, 0 und 1 annehmen.

Jetzt möchte ich ein Tensorprodukt berechnen. Nun, hier mache ich vielleicht schon einen Fehler, aber ich muss natürlich eine Basis wählen. Könnte man das sagen | M S = 1 = ( 1 0 0 ) , | M S = 0 = ( 0 1 0 ) , | M S = 1 = ( 0 0 1 ) , | M ICH = 1 = ( 1 0 0 ) , | M ICH = 0 = ( 0 1 0 ) , | M ICH = 1 = ( 0 0 1 )

Ich nehme an, dass dies bereits der Punkt ist, an dem die Dinge schief gehen, da ich vielleicht nicht die gleichen Basisvektoren für zwei Partikel auswählen kann? Wie dem auch sei, WENN obiges zutrifft, würde ich dann zum Beispiel gerne rechnen

1 2 ( | M S = 0 ich | M S = 1 ) | M ICH = 1

So wie ich das jetzt sehe, wäre das einfach ein 9x1 Vektor gleich

( 0 0 0 0 0 1 2 0 0 ich 2 )

Allerdings bin ich mir nicht sicher, ob das, was ich hier schreibe, richtig ist. Im nächsten Teil meiner Berechnungen führe ich einen hamiltonschen Term ein, der bewirkt, dass sich der Spin des ersten Teilchens in Abhängigkeit vom Spin des zweiten Teilchens entwickelt, und wenn ich die Terme ausarbeite, scheint es, als ob die Dinge schrecklich schief gehen. Da der Hamiltonian einfach eine Konstante multipliziert mit dem Tensorprodukt der beiden Z-Pauli-Spinmatrizen für ein Teilchen mit Spin 1 ist, kann da nicht viel schief gehen, also dachte ich, der Fehler muss hier irgendwo sein.

Hm, kurze Anmerkung, mir ist gerade aufgefallen, dass ich etwas falsch gemacht habe in dem Teil, wo nicht viel schief gehen konnte. Vielleicht kann ich den Beitrag löschen.
Noch besser: Lösung posten.

Antworten (1)

Die zwei Teilchen M S Und M ICH in verschiedenen Vektorräumen leben, also wählen Sie eigentlich nicht die gleichen Basisvektoren (weil die Basisvektoren der verschiedenen Partikel zu zwei getrennten Vektorräumen gehören).

Zweitens bildet das Tensorprodukt zwischen den Basisvektoren der beiden unterschiedlichen Vektorräume die Basisvektoren eines neuen 3 × 3 = 9 dimensionaler Vektorraum. Zum Beispiel:

| M S = 1 | M ICH = 0 = ( 0 1 0 0 0 0 0 0 0 )
Die Gleichung, die Sie aufgeschrieben haben, ist also richtig.

Ich würde diese kurzen Notizen dringend empfehlen, um ein besseres Gefühl für die gesamte Maschinerie zu bekommen.

Bearbeiten als Antwort auf Kommentar:

Wir müssen das Tensorprodukt verwenden, um Drehimpulse hinzuzufügen. Stellen Sie sich der Einfachheit halber ein System mit zwei Teilchen vor, und der Spin beider Teilchen ist gleich 1 / 2 . Dann können diese Partikel auf folgende Weise kombiniert werden:

↑↑ , ↑↓ , ↓↑ , ↓↓
Das bedeutet, dass der Zwei-Teilchen-Hilbert-Raum (also der dem System entsprechende Hilbert-Raum) von vier Basisvektoren aufgespannt wird:
| S 1 , M 1 ; S 2 , M 2 | S 1 , M 1 | S 2 , M 2
Anschließend zerlegen wir, wie ich in dieser Antwort skizziert habe , dieses Tensorprodukt, um die möglichen Eigenwerte für die Größe und zu bestimmen z -Bestandteil des Gesamtsystems (dh des Systems aus den beiden Spin- 1 / 2 Partikel) sein können.

Ich bin sicher, dass mir etwas fehlt, aber fügen wir nicht Drehimpuls-Eigenzustände hinzu? Sollte die Antwort also nicht eine lineare Kombination von Zuständen sein, gewichtet mit ihren Clebsch-Gordan-Koeffizienten?
@ JeffDror Ich habe meine Nachricht aktualisiert. Hoffe das beantwortet deine Frage (grob).
@JeffDror auch, das OP bittet nicht direkt darum, den Drehimpuls der beiden Teilchen hinzuzufügen.
Vielen Dank für die Hilfe und auch besonders für diese Anmerkungen, sehr hilfreich!
@ user129412 kein Problem. Das Tensorprodukt kam mir immer ein bisschen magisch vor, bis ich diese Notizen las. Daher freue ich mich zu hören, dass Sie sie auch hilfreich fanden.